- Fri Jan 21, 2011 12:00 am
#27911
Complete Question Explanation
Flaw in the Reasoning. The correct answer choice is (E)
In this stimulus, a columnist discusses recent research suggesting that vigorous exercise can significantly lower one’s chances for certain cardio-respiratory diseases. The columnist then concludes that one should ignore older studies that claim the same effect could be achieved by non-strenuous walking.
The question stem asks for the vulnerability in columnist’s reasoning. The columnist draws a conclusion based on the mistaken notion that because benefits can be derived in one manner, the same benefits can’t be achieved by other means:
Answer choice (B): If the discussion were about overall health, this could be seen as weakening the columnist’s conclusion, but this discussion is only about the effect of exercise on cardio-respiratory diseases and whether that exercise must be vigorous. The fact that vigorous exercise may bring some outside risk does not play into an argument about its effects on the certain cardiovascular respiratory illnesses referenced in the stimulus.
Answer choice (C): Overlooking another possible benefit of vigorous exercise is not a flaw in this case. The discussion in the stimulus only concerns the beneficial effects of exercise on cardio-respiratory diseases and whether such exercise must be vigorous.
Answer choice (D): The discussion in the stimulus is not about perceptions of health, and we could only speculate as to the relationship between perception and physical manifestation. In any case, people who perceive themselves as healthy can still acquire cardio-respiratory diseases, and the columnist’s conclusion involves the effect of exercise. Since this answer choice does not discuss exercise at all, it does not reflect the flaw in the author’s reasoning.
Answer choice (E): This is the correct answer choice. The argument is weak because it fails to show that the conclusion of the recent report is better justified than an opposing conclusion reached in older studies. This mirrors our prephrased answer; no logical justification is offered for ignoring the old studies in favor of the new report.
Flaw in the Reasoning. The correct answer choice is (E)
In this stimulus, a columnist discusses recent research suggesting that vigorous exercise can significantly lower one’s chances for certain cardio-respiratory diseases. The columnist then concludes that one should ignore older studies that claim the same effect could be achieved by non-strenuous walking.
The question stem asks for the vulnerability in columnist’s reasoning. The columnist draws a conclusion based on the mistaken notion that because benefits can be derived in one manner, the same benefits can’t be achieved by other means:
- Premise: Strenuous walking reduces one changes of getting certain diseases.
Flawed Conclusion: Non-strenuous walking must not be effective.
Answer choice (B): If the discussion were about overall health, this could be seen as weakening the columnist’s conclusion, but this discussion is only about the effect of exercise on cardio-respiratory diseases and whether that exercise must be vigorous. The fact that vigorous exercise may bring some outside risk does not play into an argument about its effects on the certain cardiovascular respiratory illnesses referenced in the stimulus.
Answer choice (C): Overlooking another possible benefit of vigorous exercise is not a flaw in this case. The discussion in the stimulus only concerns the beneficial effects of exercise on cardio-respiratory diseases and whether such exercise must be vigorous.
Answer choice (D): The discussion in the stimulus is not about perceptions of health, and we could only speculate as to the relationship between perception and physical manifestation. In any case, people who perceive themselves as healthy can still acquire cardio-respiratory diseases, and the columnist’s conclusion involves the effect of exercise. Since this answer choice does not discuss exercise at all, it does not reflect the flaw in the author’s reasoning.
Answer choice (E): This is the correct answer choice. The argument is weak because it fails to show that the conclusion of the recent report is better justified than an opposing conclusion reached in older studies. This mirrors our prephrased answer; no logical justification is offered for ignoring the old studies in favor of the new report.